K
Khách

Hãy nhập câu hỏi của bạn vào đây, nếu là tài khoản VIP, bạn sẽ được ưu tiên trả lời.

16 tháng 2 2020

Áp dụng bđt Cauchy - Schwarz dạng Engel:

\(VT=\frac{1}{4a}+\frac{4}{4b}+\frac{4}{4c}\ge\frac{\left(1+2+2\right)^2}{4\left(a+b+c\right)}=\frac{25}{4}\)

(Dấu "=" xảy ra khi \(a=\frac{1}{5};b=c=\frac{2}{5}\))

Ai muốn vào team tui không

Xin lỗi rất nhiều vì đã làm sai quy luật, nội quy ạ

Mong mọi người đừng chửi

Học Tốt

30 tháng 4 2015

bạn xem bài này tại đây: 

http://d.violet.vn/uploads/resources/615/2779702/preview.swf

22 tháng 4 2021

bài hơi khoai

22 tháng 4 2021

Không mất tính tổng quát giả sử \(c=max\left\{a,b,c\right\}\)

\(\Rightarrow2c\ge a+b\)

\(\Rightarrow c\ge\frac{a+b}{2}\)

Từ giả thiết \(\Rightarrow a,b\le1\)

\(\Rightarrow ab\le1\)( *)

Đặt \(P=\frac{1}{a+b}+\frac{1}{b+c}+\frac{1}{c+a}-\frac{5}{2}\)

\(=\frac{1}{a+b}+\frac{1}{b+\frac{1-ab}{a+b}}+\frac{1}{a+\frac{1-ab}{a+b}}-\frac{5}{2}\)

Đặt \(S=\frac{1}{a+b+\frac{1}{a+b}}+a+b+\frac{1}{a+b}-\frac{5}{2}\)

Xét hiệu \(P-S=\)\(\frac{1}{a+b}+\frac{1}{b+\frac{1-ab}{a+b}}+\frac{1}{a+\frac{1-ab}{a+b}}-\frac{5}{2}-\)\(-\frac{1}{a+b+\frac{1}{a+b}}-a-b-\frac{1}{a+b}+\frac{5}{2}\)

\(=\frac{1}{\frac{ab+b^2+1-ab}{a+b}}+\frac{1}{\frac{a^2+ab+1-ab}{a+b}}-\frac{1}{\frac{\left(a+\right)^2+1}{a+b}}-\left(a+b\right)\)

\(=\frac{a+b}{b^2+1}+\frac{a+b}{c^2+1}-\left(a+b\right)\left[1+\frac{1}{1+\left(a+b\right)^2}\right]\)

Ta sẽ chứng minh \(\frac{a+b}{b^2+1}+\frac{a+b}{c^2+1}-\left(a+b\right)\left[1+\frac{1}{1+\left(a+b\right)^2}\right]\ge0\)

\(\Leftrightarrow\frac{a+b}{b^2+1}+\frac{a+b}{c^2+1}\ge\left(a+b\right)\left[1+\frac{1}{1+\left(a+b\right)^2}\right]\)

\(\Leftrightarrow\frac{1}{b^2+1}+\frac{1}{c^2+1}\ge1+\frac{1}{1+\left(a+b\right)^2}\)

\(\Leftrightarrow\frac{2+a^2+b^2}{\left(1+a^2\right)\left(1+b^2\right)}\ge\frac{2+\left(a+b\right)^2}{1+\left(a+b\right)^2}\)

\(\Rightarrow\left(2+b^2+a^2\right)\left[1+\left(a+b\right)^2\right]\ge\left[2+\left(a+b\right)^2\right]\left(1+a^2\right)\left(1+b^2\right)\)

\(\Leftrightarrow2+2\left(a+b\right)^2+\left(a+b\right)^2\left(a^2+b^2\right)+a^2+b^2\ge\left[2+\left(a+b\right)^2\right]\left(1+a^2+b^2+a^2b^2\right)\)

\(\Leftrightarrow2+2\left(a+b\right)^2+\left(a+b\right)^2\left(a^2+b^2\right)+a^2+b^2-2a^2b^2-\left(a+b\right)^2\left(a^2+b^2\right)-\left(a+b\right)^2a^2b^2\)\(-2-2\left(a^2+b^2\right)-\left(a+b^2\right)\ge0\)

\(\Leftrightarrow-2a^2b^2-\left(a+b\right)^2a^2b^2+a^2+b^2-\left(a+b\right)^2\ge0\)

\(\Leftrightarrow ab\left[ab\left(a+b\right)^2+2ab-2\right]\le0\)

\(\Leftrightarrow ab\left(a+b\right)^2+2ab-2\le0\)( do a,b \(\ge0\))

\(\Leftrightarrow ab\left(a+b\right)^2\le2\left(1-ab\right)\)

\(\Leftrightarrow ab\left(a+b\right)^2\le2c\left(a+b\right)\) (1)

Mà \(c\ge\frac{a+b}{2}\)

\(\Rightarrow2c\left(a+b\right)\ge\left(a+b\right)^2\)

Ta có: \(\left(a+b\right)^2\ge ab\left(a+b\right)^2\)

\(\Leftrightarrow\left(a+b\right)^2\left(1-ab\right)\ge0\)( đúng do (*) ) 

\(\Rightarrow\left(1\right)\)đúng

\(\Rightarrow P-S\ge0\)

\(\Rightarrow P\ge S\)

Ta phải chứng minh \(S\ge0\)

\(\Leftrightarrow\frac{1}{a+b+\frac{1}{a+b}}+a+b+\frac{1}{a+b}\ge\frac{5}{2}\)

\(\Leftrightarrow\frac{a+b}{1+\left(a+b\right)^2}+\frac{1+\left(a+b\right)^2}{a+b}\ge\frac{5}{2}\) (2) 

Đặt \(x=\frac{1+\left(a+b\right)^2}{a+b}\)

Ta có: \(1+\left(a+b\right)^2\ge2\left(a+b\right)\)

\(\Leftrightarrow\left(a+b-1\right)^2\ge0\)( đúng )

\(\Rightarrow x=\frac{1+\left(a+b\right)^2}{a+b}\ge2\)

=> (2) có dạng \(x+\frac{1}{x}\ge\frac{5}{2}\)

\(\Leftrightarrow2x^2-5x+2\ge0\)

\(\Leftrightarrow\left(2x-1\right)\left(x-2\right)\ge0\)( đúng )

\(\Rightarrow S\ge0\)mà \(P\ge S\)

\(\Rightarrow P\ge0\)

\(\Leftrightarrow\frac{1}{a+b}+\frac{1}{b+c}+\frac{1}{c+a}\ge\frac{5}{2}\)

Dấu "=" xảy ra \(\Leftrightarrow\hept{\begin{cases}a+b=1\\ab+bc+ca=1\\ab\left[ab\left(a+b\right)^2+2ab-2\right]=0\end{cases}}\)

\(\Leftrightarrow\orbr{\begin{cases}a=c=1;b=0\\b=c=1;a=0\end{cases}}\)

17 tháng 2 2020

https://olm.vn/hoi-dap/detail/239526218296.html

27 tháng 2 2020

Sử dụng phân tích tuyệt vời của Ji Chen:

\(VT-VP=\frac{4\left(a+b+c-2\right)^2+abc+3\Sigma a\left(b+c-1\right)^2}{\left(a+b\right)\left(b+c\right)\left(c+a\right)}\ge0\)

15 tháng 9 2020

Ta dễ có:\(\frac{1}{a^2+1}=\frac{a^2+1-a^2}{a^2+1}=1-\frac{a^2}{a^2+1}\ge1-\frac{a^2}{2a}=1-\frac{a}{2}\)

Một cách tương tự \(\frac{1}{b^2+1}\ge1-\frac{b}{2};\frac{1}{c^2+1}\ge1-\frac{c}{2}\)

Khi đó: \(\frac{1}{a^2+1}+\frac{1}{b^2+1}+\frac{1}{c^2+1}\ge3-\frac{a+b+c}{2}\)

Cần chứng minh: \(3-\frac{a+b+c}{2}\ge\frac{3}{2}\Leftrightarrow a+b+c\le3\)

Hình như có gì đó sai sai @@

15 tháng 9 2020

Lời giải kia sai rồi :V Làm cách khác:

Ta có:\(\frac{1}{a^2+1}=\frac{a^2+1-a^2}{a^2+1}=1-\frac{a^2}{a^2+1}\)

Tương tự rồi ta được:

\(LHS=3-\left(\frac{a^2}{a^2+1}+\frac{b^2}{b^2+1}+\frac{c^2}{c^2+1}\right)\)

Bất đẳng thức cần chứng minh tương đương với: 

\(\frac{a^2}{a^2+1}+\frac{b^2}{b^2+1}+\frac{c^2}{c^2+1}\le\frac{3}{2}\)

\(\Leftrightarrow\frac{a^2}{3a^2+3}+\frac{b^2}{3b^2+3}+\frac{c^2}{3c^2+3}\le\frac{1}{2}\)

Ta dễ có được:

\(\frac{4a^2}{3a^2+3}=\frac{4a^2}{3a^2+ab+bc+ca}=\frac{\left(a+a\right)^2}{a\left(a+b+c\right)+2a^2+bc}\le\frac{a^2}{a\left(a+b+c\right)}+\frac{a^2}{2a^2+bc}\)

Tương tự:

\(\frac{4b^2}{3b^2+3}\le\frac{b^2}{b\left(a+b+c\right)}+\frac{b^2}{2b^2+ca};\frac{4c^2}{3c^2+3}\le\frac{c^2}{c\left(a+b+c\right)}+\frac{c^2}{2c^2+ab}\)

\(\Rightarrow LHS\le\frac{1}{4}\left(\frac{a}{a+b+c}+\frac{b}{a+b+c}+\frac{c}{a+b+c}+\Sigma\frac{a^2}{2a^2+bc}\right)=\frac{1}{4}\left(1+\Sigma\frac{a^2}{2a^2+bc}\right)\)

Một cách khác ta dễ có được: \(\Sigma\frac{a^2}{2a^2+bc}\le1\)

Done !

17 tháng 12 2022

không mất tính tổng quát giả sử  $a\leqslant b\leqslant c$

đặt 

x=a+b+c

y=ab+bc+ac

z=abc

ta có bđt thức đầu tiên sẽ tương đương với 

$(x+3a)(x+3b)(x+3c)> 25(x-a)(x-b)(x-c)$

 

$\Leftrightarrow x^{3}+3x^{2}(a+b+c)+9x(ab+bc+ac)+27abc> 25(x^{3}-x^{2}(a+b+c)+x(ab+bc+ac)-abc)$

 

$\Leftrightarrow x^{3}-4xy+13z> 0$ (1)

đặt S=VT

ta có

S=$(a+b+c)^{3}-4(a+b+c)(ab+bc+ac)+13abc=(a+b+c)((a+b+c)^{2}-4(ab+bc+ac))+13abc=(a+b+c)((a+b-c)^{2}-4ab)+13abc= (a+b+c)(a+b-c)^{2}+ab(9c-4b-4c)$

vậy (1) tương đương với

$(a+b+c)(a+b-c)^{2}+ab(9c-4b-4c)> 0$

do $0< a\leqslant b\leqslant c$

nên bđt trên hiển nhiên đúng 

vậy được đpcm

24 tháng 4 2017

Nhân 2 vế của \(\frac{1}{a}+\frac{1}{b}+\frac{1}{c}=1\) có: \(ab+bc+ca=abc\)

Ta có: 

\(\frac{a^2}{a+bc}=\frac{a^3}{a^2+abc}=\frac{a^3}{a^2+ab+bc+ca}=\frac{a^3}{\left(a+b\right)\left(a+c\right)}\)

Áp dụng BĐT AM-GM ta có:

\(\frac{a^2}{a+bc}=\frac{a^3}{\left(a+b\right)\left(a+c\right)}+\frac{a+b}{8}+\frac{a+c}{8}\)

\(\ge3\sqrt[3]{\frac{a^3}{\left(a+b\right)\left(a+c\right)}\cdot\frac{a+b}{8}\cdot\frac{a+c}{8}}=\frac{3a}{4}\)

Tương tự cho 2 BĐT còn lại ta có:

\(\frac{b^2}{b+ca}+\frac{a+b}{8}+\frac{b+c}{8}\ge\frac{3b}{4};\frac{c^2}{c+ab}+\frac{a+c}{8}+\frac{b+c}{8}\ge\frac{3c}{4}\)

Cộng theo vế 3 BĐT trên ta có:

\(VT+\frac{4\left(a+b+c\right)}{8}\ge\frac{3\left(a+b+c\right)}{4}\)

\(\Leftrightarrow VT+\frac{4\left(a+b+c\right)}{8}\ge\frac{6\left(a+b+c\right)}{8}\)

\(\Leftrightarrow VT\ge\frac{a+b+c}{4}=VP\). Ta có ĐPCM

Áp dụng AM-GM ta có : \(\frac{a}{a^2+1}=\frac{a}{a^2+\frac{1}{9}+\frac{8}{9}}\le\frac{a}{\frac{2a}{3}+\frac{8}{9}}=\frac{9a}{6a+8}\)

Áp dụng BĐT : \(\left(x+y+z\right)\left(\frac{1}{x}+\frac{1}{y}+\frac{1}{z}\right)\ge9\)với \(x,y,z>0\)( Dễ dàng CM bằng AM-GM )

\(\left(6a+8+6b+8+6c+8\right)\left(\frac{1}{6a+8}+\frac{1}{6b+8}+\frac{1}{6c+8}\right)\ge9\)

\(\frac{1}{6a+8}+\frac{1}{6b+8}+\frac{1}{6c+8}\ge\frac{9}{30}=\frac{3}{10}\)

Ta có : \(\frac{9a}{6a+8}=\frac{3}{2}-\frac{12}{6a+8}\)

\(\rightarrow\frac{9a}{6a+8}+\frac{9b}{6b+8}+\frac{9c}{6c+8}=\frac{9}{2}-12\left(\frac{1}{6a+8}+\frac{1}{6b+8}+\frac{1}{6c+8}\right)\)

Lại có : \(\frac{9}{2}-12\left(\frac{1}{6a+8}+\frac{1}{6b+8}+\frac{1}{6c+8}\right)\le\frac{9}{2}-12.\frac{3}{10}=\frac{9}{2}-\frac{18}{5}=\frac{9}{10}\)

24 tháng 5 2021

Các bạn giúp mình với !

28 tháng 12 2016

\(\frac{a}{1+b^2}=a-\frac{ab^2}{1+b^2}\ge a-\frac{ab^2}{2b}=a-\frac{ab}{2}\)

Tương tự : \(\frac{b}{1+c^2}\ge b-\frac{bc}{2}\) ; \(\frac{c}{1+a^2}\ge c-\frac{ac}{2}\)

Cộng theo vế : \(\frac{a}{1+b^2}+\frac{b}{1+c^2}+\frac{c}{1+a^2}\ge3-\frac{1}{2}\left(ab+bc+ac\right)\ge3-\frac{1}{2}.\frac{\left(a+b+c\right)^2}{3}=\frac{3}{2}\)

\(\Rightarrow\frac{a}{1+b^2}+\frac{b}{1+c^2}+\frac{c}{1+a^2}\ge\frac{3}{2}\)